Inscription / Connexion Nouveau Sujet
Niveau maths sup
Partager :

PROBLEME : Champ E et B

Posté par
pfff
08-04-21 à 15:26

Bonjour, j'aimerais de l'aide afin de pouvoir terminer ce problème. Merci

PROBLEME

A- Première partie : Préliminaires

1/ Donner l'expression de la force \vec{f} qui s'exerce sur une particule de charge q se déplaçant à une vitesse \vec{v} dans un référentiel galiléen lorsqu'elle est soumise simultanément à un champ électrique \vec{E} et un champ magnétique \vec{B}.
.
2/ Action d'un champ magnétique :
En appliquant le théorème de l'énergie cinétique ou le théorème de la puissance cinétique, montrer qu'un champ magnétique \vec{B} appliqué seul à une particule chargée ne peut pas lui communiquer d'énergie.


B- Deuxième partie : Accélération d'une particule soumise à un champ électrique

On se place en l'absence de champ magnétique et on négligera l'effet de la pesanteur sur la particule étudiée.

3./ Une particule de charge q est soumise à un champ électrique \vec{E} , stationnaire et uniforme, créé par deux plaques A et D parallèles et respectivement portées à un potentiel V_A nul et à un potentiel V_D non nul. La particule est initialement issue de la plaque A avec une vitesse nulle et se dirige vers la plaque D.

3.1/ Quel doit être le signe du potentiel V_D de la plaque D pour que la particule soit accélérée vers D dans le cas où la particule a une charge q négative ?

3.2/ Calculer en fonction de q, V_D et m, par rapport au référentiel d'étude considéré comme galiléen, la vitesse \vec{v_D} de la particule lorsqu'elle atteint la plaque D.

3.3/ Citer quelques appareils dans lesquels on utilise un tel dispositif d'accélération de particules par un champ électrique.

3.4/ Quelle est l'énergie cinétique acquise par un électron (de charge -e) accéléré sous une différence de potentiel de 1V ? On exprimera le résultat en Joule et en électron-volt.

On donne : charge électrique élémentaire : e = 1,6.10^{-19} C


C- Troisième partie : Mouvement d'une particule chargée dans un champ magnétique

On étudie maintenant le mouvement d'une particule chargée de charge q et de masse m dans un champ magnétique \vec{B} uniforme et stationnaire.
On se place dans un référentiel d'étude galiléen, rapporté à un repère orthonormé Oxyz .
Le champ magnétique \vec{B} = B\vec{e_z} est dirigé suivant l'axe Oz (on considèrera B > 0 ).

On considèrera uniquement l'effet de la force magnétique et on négligera l'effet de la pesanteur.

On pose : w_o = \frac{qB}{m}

4/ On se place dans un premier temps dans le cas où à l'instant initial (t = 0) la particule est à l'origine O du repère et la vitesse initiale de la particule est dirigée suivant l'axe Ox ( \vec{v_o} = v_o\vec{e_x} \: avec v_o \succ 0 ).

4.1/ En prenant en compte les conditions initiales, montrer que la trajectoire de la particule est plane et contenue dans le plan Oxy.

4.2/ Montrer que le module de la vitesse de la particule est constant.

5/ On se place maintenant dans le cas où la vitesse initiale \vec{v_o} fait un angle \alpha_o avec l'axe Oz et l'on choisit les deux axes Ox et Oy de telle façon que le vecteur \vec{v_o} soit contenu dans le plan xOz.
(\vec{v_o} = v_o_x\vec{e_x} + v_o_z\vec{e_z} \: avec\: v_o_x \succ 0 \: et\: v_{oz} \succ 0 )


PROBLEME : Champ E et B

5.1/ Par projection de l'équation vectorielle qui résulte de l'application du principe fondamental de la dynamique sur les trois axes du repère, déterminer les trois équations différentielles qui régissent les coordonnées x, y et z de la particule et leurs dérivées par rapport au temps.

5.2/ Dans deux des équations différentielles précédentes, les variables x et y sont couplées (x et y sont présentes simultanément dans chacune des deux équations).
Par intégration directe des équations différentielles couplées précédentes et en utilisant les conditions initiales, trouver une première équation liant \frac{dx}{dt} et y et une deuxième équation liant \frac{dy}{dt} et x.

5.3/ Déduire des deux questions précédentes les deux équations différentielles du second ordre découplées en x et en y.

5.4/ Par intégration des trois équations différentielles découplées en x, y et z, établir les équations horaires x(t), y(t) et z(t) (On détaillera le calcul pour chaque équation différentielle et notamment la prise en compte des conditions initiales).

5.5/ Montrer que la trajectoire dans le plan Oxy est un cercle dont on précisera le centre, le rayon et la période.

5.5/ La trajectoire est une hélice. Déterminer son pas.


ELEMENTS DE REPONSES

A- Première partie : Préliminaires

1-\vec{f} = q\vec{E} + q\vec{v}\Lambda \vec{B}

2- Je ne vois pas ce qu'il faut montrer

B- Deuxième partie : Accélération d'une particule soumise à un champ électrique

3-1- La particule de charge q négative est accéléré de A vers D, alors la plaque A est négative et la  plaque D est chargé positivement.

3-2- j'obtiens v_D = \sqrt{\frac{-2qV_D}{m}}

3-3- Je n'ai pas pu trouver

3-4- Ec = -qV_D
AN : Ec = 1,6.10^-^1^9 J = 1 ev


C- Troisième partie : Mouvement d'une particule chargée dans un champ magnétique

4-1- Je n'arrive pas à trouver

4-2- Je  n'arrive pas à trouver

5-1-

d'après le PFD \sum{\vec{F_{ext}}} = m\vec{a} \Rightarrow e\vec{v} \Lambda \vec{B} = m\vec{a}

m\vec{a} = \begin{cases} & \text{ } m\ddot{x} \\ & \text{ } m\ddot{y} \\ & \text{ } m\ddot{z} \end{cases}

e\vec{v} \Lambda \vec{B} = \begin{cases} & \text{ } \dot{yBe} \\ & \text{ } \dot{x}Be \\ & \text{ } 0 \end{cases}

on obtient :

\begin{cases} & \text{ } m\ddot{x} = \dot{yBe} \\ & \text{ } m\ddot{y} = \dot{x}Be \\ & \text{ } m\ddot{z} = 0 \end{cases}


5-2- Par intégration directe on obtient :

 \begin{cases} & \text{ } m\dot{x} = eyB + C \\ & \text{ } \\ & \text{ } m\dot{y} = exB + C' \end{cases}

initialement \dot{x}(t=0) = v_o_x \: et \: y(t=0) = 0 donc  C = mv_o_x

initialement \dot{y}(t=0) = 0 \: et \: y(t=0) = 0 donc  C' = 0

finalement on a :

 = \begin{cases} & \text{ } m\frac{dx}{dt} = eyB + mv_o_x \\ & \text{ } \\ & \text{ } m\frac{dy}{dt} = exB \end{cases}

5-4- Je ne comprends pas cette question

5-5-/5-6/ je n'ai pas pu faire

Posté par
vanoise
re : PROBLEME : Champ E et B 08-04-21 à 15:35

Bonjour
Pour la question 2 : tu as fait la démonstration dans l'autre exercice dont je me suis occupé.

Posté par
vanoise
re : PROBLEME : Champ E et B 08-04-21 à 15:42

3.3 ;
Les appareils numériques actuels utilisent de moins en moins les "canons à électrons". Ces dispositifs étaient présents dans les oscilloscopes et les postes de télévisions anciens, les anciens moniteurs d'ordinateur, avant la technologie des écrans plats.

Posté par
pfff
re : PROBLEME : Champ E et B 08-04-21 à 15:48

2- oui mais cette fois on dit avec le TEC  comment je fais ?

Posté par
vanoise
re : PROBLEME : Champ E et B 08-04-21 à 16:07

Ne pas oublier que le travail entre deux instants quelconques de date zéro et t1 s'obtient en intégrant l'expression en fonction du temps de la puissance instantanée. Le théorème de l'énergie cinétique, en négligeant le travail du poids s'écrit simplement :

\frac{1}{2}m.v_{(t_{1})}^{2}-\frac{1}{2}m.v_{o}^{2}=W=\int_{0}^{t_{1}}p_{(t)}.dt

Or, tu as montré hier : p_{(t)}=0\;\forall t ...

3.4 : OK mais juste un détail : eV plutôt que ev pour le symbole de l'unité.

Citation :
4-1- Je n'arrive pas à trouver

4-2- Je n'arrive pas à trouver


Cela a été traité dans l'autre exercice ! Pas très motivant pour moi qui t'ai aidé hier !

Pour la suite, l'écriture est allégée en posant comme demandé par l'énoncé : w_o = \frac{qB}{m} .

Posté par
pfff
re : PROBLEME : Champ E et B 08-04-21 à 16:20

2-2- je suis aussi parti sur le fait que

\Delta EC = \sum{W(\vec{F}}) = q\vec{v}\Lambda \vec{B} or \vec{v}\perp \vec{B}

Donc \Delta EC = 0

4-1- oui mais ici on a pas \vec{a} \perp \vec{B}

Posté par
vanoise
re : PROBLEME : Champ E et B 08-04-21 à 16:50

Par rapport à l'étude d'hier, la vitesse initiale n'est plus perpendiculaire au vecteur \vec B ; le mouvement n'est plus un mouvement circulaire uniforme mais un mouvement hélicoïdal (j'anticipe un peu le résultat final ). Le fait de modifier la direction du vecteur vitesse initiale ne modifie en rien l'expression du vecteur force et donc aussi l'expression du vecteur accélération.
Propriété évidente du produit vectoriel : \vec{a} \perp \vec{B}

Posté par
pfff
re : PROBLEME : Champ E et B 08-04-21 à 20:27

Ils ont aussi dit d'utiliser les conditions initiales donc on a :

\vec{OM}_O \begin{cases} & \text{ } x_0= 0 \\ & \text{ } y_0= 0 \\ & \text{ } z_0= 0 \end{cases}                                  \vec{v}_O \begin{cases} & \text{ } v_0_x= v_0 \\ & \text{ } v_0_y= 0 \\ & \text{ } v_0_z= 0 \end{cases}                        \vec{a} \begin{cases} & \text{ } a_x= 0 \\ & \text{ } a_y= a \\ & \text{ } a_z= 0 \end{cases}

\vec{v} \begin{cases} & \text{ } v_x= v_0\\ & \text{ } v_y= at \\ & \text{ } v_z= 0 \end{cases}                                      \vec{OM} \begin{cases} & \text{ } x= v_0t\\ & \text{ } y= \frac{1}{2}at² \\ & \text{ } z= 0 \end{cases}

On en déduit que le mouvement est plan et contenue dans le plan Oxy


Ensuite comment démontrer  que le module de la vitesse est constante sans utiliser la base de Frenet ?

Posté par
vanoise
re : PROBLEME : Champ E et B 08-04-21 à 21:14

Je considère 4.2 comme réglée : la réponse a déjà été fournie à la question 2.
Pour 4.1 : ne pas oublier l'expression de l'accélération :

\overrightarrow{a}=\frac{q}{m}\cdot\overrightarrow{v}\wedge\overrightarrow{B}
Cela implique :

\overrightarrow{a}\bot\overrightarrow{B}   donc :  a_{z}=0
Ce que tu as écrit concernant az puis vz pour aboutir à z=0 est correct mais attention : ce que tu as écrit concernant x et y est faux : tu as montré hier que le mouvement était circulaire uniforme  dans le plan (Oxy) !

Posté par
pfff
re : PROBLEME : Champ E et B 08-04-21 à 21:19

mais ce sont bien les conditions initiales que j'ai utilisé pour x et y aussi , je ne vois pas encore l'erreur

Posté par
pfff
re : PROBLEME : Champ E et B 08-04-21 à 21:21

effectivement d'après la question 2, on en déduit la question 4-2

Posté par
vanoise
re : PROBLEME : Champ E et B 08-04-21 à 21:31

Pour x et y, la méthode correcte serait celle que tu as utilisée en 5.1 mais  cela est inutile  : la question 4 demande juste de démontrer que la particule se déplace dans le plan ( Oxy).

Posté par
pfff
re : PROBLEME : Champ E et B 08-04-21 à 21:41

ah ok donc il faut juste montrer que z=0. OK

Posté par
pfff
re : PROBLEME : Champ E et B 08-04-21 à 21:45

Si mes réponses de 5-1 à 5-2 sont correctes. Comment dois-je procéder pour faire la suite

Posté par
vanoise
re : PROBLEME : Champ E et B 08-04-21 à 23:20

Concernant 5, tu es bien parti :

Concernant z(t) : puisque az=0, on obtient immédiatement :

z_{(t)}=v_{oz}.t=v_{o}.\cos\left(\alpha_{o}\right).t

Concernant, x(t) et y(t) j'avais oublié de signaler une faute de signe ; en utilisant les notations de l'énoncé, on obtient :

\begin{cases}
 \\ \ddot{x}=\omega_{o}.\dot{y} & (1)\\
 \\ \ddot{y}=-\omega_{o}.\dot{x} & (2)\\
 \\ \dot{x}=\omega_{o}.y+v_{o}.\sin\left(\alpha_{o}\right) & (3)\\
 \\ \dot{y}=-\omega_{o}.x & (4)
 \\ \end{cases}

En remplaçant dans (1)  \dot{y} par son expression obtenue en (4) : tu obtiens une équation différentielle très simple vérifiée par x(t) dont tu connais sûrement la solution. De même en remplaçant dans (2)  \dot{x} par son expression obtenue en 3, tu obtiens une autre équation différentielle du second ordre vérifiée par y(t) facile à résoudre.

Pour 5.5 : n'oublie pas la relation entre le carré du sinus et le carré du cosinus d'un même angle....

Posté par
pfff
re : PROBLEME : Champ E et B 09-04-21 à 01:17

voici ce que je trouve finalement pour la question 5-4-

x(t) = \frac{v_0}{\omega_0}sin\alpha _0\, sin(\omega _0t)

y(t) = \frac{v_0}{\omega_0}sin\alpha _0\, cos(\omega _0t) - \frac{v_0}{\omega_0}sin\alpha _0

et bien sur z(t) = v_0cos\alpha _0t

normalement pour trouver l'équation du cercle je dois faire x(t)² + y(t)² mais avec l'expression de y(t ) c'est un peu difficile

Posté par
vanoise
re : PROBLEME : Champ E et B 09-04-21 à 11:52

Pourquoi ne pas écrire :

\begin{cases}
 \\ x_{(t)}=\frac{v_{o}.\sin\left(\alpha_{o}\right)}{\omega_{o}}\cdot\sin\left(\omega_{o}.t\right)\\
 \\ y_{(t)}+\frac{v_{o}.\sin\left(\alpha_{o}\right)}{\omega_{o}}=\frac{v_{o}.\sin\left(\alpha_{o}\right)}{\omega_{o}}\cdot\cos\left(\omega_{o}.t\right)
 \\ \end{cases}

avant d'élever au carré ?

je me pose une question à propos de l'énoncé. La partie II précise que la particule est un électron de charge q=-e. Les parties sont-elles indépendantes où au contraire faut-il considérer dans cette troisième partie qu'il s'agit toujours d'un électron ? Si oui : puisque :

\omega_{o}=\frac{qB}{m}=-\frac{e.B}{m}<0

on pourrait poser (notation choisie par pur hasard évidemment ) :

R=-\frac{v_{o}.\sin\left(\alpha_{o}\right)}{\omega_{o}}=\frac{m.v_{o}.\sin\left(\alpha_{o}\right)}{e.B}

Posté par
pfff
re : PROBLEME : Champ E et B 09-04-21 à 14:16

non la partie 2) considère toujours une particule de charge q seulement que pour la question 3-4), voulant faire une application numérique on a  donc considéré un électron.

Sinon l'exercice travaille avec une particule de charge q

Posté par
pfff
re : PROBLEME : Champ E et B 09-04-21 à 14:25

donc si je veux considérer R = \frac{v_o}{\omega _0}sin\alpha _0

j'obtiens R² = x² + ( y + R )² \Rightarrow \frac{x²}{R²} + \frac{(y+R)²}{R²} = 1

donc la trajectoire est un cercle de centre ( 0 , -R ) de rayon R

la période je ne vois pas

Posté par
vanoise
re : PROBLEME : Champ E et B 09-04-21 à 18:39

Citation :
donc la trajectoire est un cercle de centre  ( 0 , -R ) de rayon R

Si le signe de q est inconnu, celui de R est aussi inconnu. D'accord avec les coordonnée du centre de la trajectoire du projeté orthogonal de la particule dans le plan (Oxy) mais il faut considérer le rayon du cercle (rayon nécessairement positif) comme égal à |R|.
La période est le plus petit intervalle de temps qui permet de retrouver les mêmes coordonnées x et y.
cos(o.(t+T))=cos(o.t)
sin(o.(t+T))=sin(o.t)
Cela implique :
|o|.T=2 (en radians)

Posté par
praf
re : PROBLEME : Champ E et B 10-04-21 à 18:04

Salut mes amis,
Merci pour ces explications interessantes,
Je ne veut pas perturber pfff mais j'ai une question à Vanoise
Dans la question 4
Si on suppose que la vitesse fait un angle alpha avec ox mais reste dans le plan oxy (reste perpondeculaire a B v=cos \alpha e_x +sin \alpha e_y)
Quel est la nature de trajectoire

Posté par
vanoise
re : PROBLEME : Champ E et B 10-04-21 à 18:24

Bonjour praf

Citation :
Quel est la nature de trajectoire

C'est justement l'objet de la question 5.5 qui n'a pas encore été abordée par pfff
Patience...

Posté par
praf
re : PROBLEME : Champ E et B 10-04-21 à 18:55

Merci Vanoise,
Dans 5.5 la condition proposé est que la vitesse fait un angle quelconque avec e_z ici la trajectoire n'est pas plan mais  hélice(combinaison d' un mouvement circulaire avec un mouvement rectiligne suivant e_z)
Moi je propose que v perpenduclaure a e_z mais pas collineaire avec e_x
Je peut devener que la tragectoire est un cercle mais avec un autre centre et un autre rayon

Posté par
vanoise
re : PROBLEME : Champ E et B 10-04-21 à 19:20

Citation :
Je peux deviner que la trajectoire est un cercle

Oui : cela a été traité très récemment ici :
Mouvement de particules chargées

Posté par
praf
re : PROBLEME : Champ E et B 11-04-21 à 10:46

Merci Vanoise



Mentions légales - Retrouvez cette page sur l'île de la physique - chimie
© digiSchool 2024

Vous devez être membre accéder à ce service...

Pas encore inscrit ?

1 compte par personne, multi-compte interdit !

Ou identifiez-vous :


Rester sur la page

Désolé, votre version d'Internet Explorer est plus que périmée ! Merci de le mettre à jour ou de télécharger Firefox ou Google Chrome pour utiliser le site. Votre ordinateur vous remerciera !